2 1/4 = how many quaters

Answers

Answer 1

Answer:

9 quarters

Step-by-step explanation:

in 2, there are 8 quarters, plus one more quarter

Answer 2
9 quarters 2 is 8 and then 8+1=9

Related Questions

Factor by using the perfect square trinomial formula
x^2+4x+4
A. x^2 +4x+4=__
B. The poly. is prime

Answers

It should be(x➕2)^2.right?

Why do we study statistics?
A. Because technological advances have introduced massive amounts of data into the world
B. Because statistics can be applied to every field of study
C. Because many of the statistical stories told in the media are misleading
D. All of the above

help me please please please please please please please please please please please please please please please please please please please please please please please please please please please please please please please ​

Answers

Answer:

B. Because statistics can be applied to every field of study

can someone answer this please
edge2021

Answers

Answer:

Step-by-step explanation:

it apperes to me that all are rational numbers

Hope this helps

A Greek mathematician named Pythagoras is said to have discovered the following number pattern. Find the next three sums by using inductive reasoning. Look for the pattern don't just add
1st row:1=1
2nd row: 1+3=4
3rd row: 1+3+5=9
4th row: 1+3+5+7=16
5th row: 1+3+5+7+9=?
6th row: 1+3+5+7+9+11=?
7th row: 1+3+5+7+9+11+13=?

Answers

5th: 25
6th: 36
7th: 49

A polynomial is
if its only factors over the integers are 1 and itself.

Answers

Answer: Prime

A polynomial is  prime    if its only factors over the integers are 1 and itself.

==============================================================

Explanation:

Think of the idea of factoring whole numbers like factoring 36 into 6*6 which further breaks down into (2*3)*(2*3) = 2^2*3^2. This shows that 36 is composite and not prime.

Something like 7 is prime because the only factors are 1 and itself.

This idea extends into polynomials as well.

Something like x^2+4x+4 factors to (x+2)(x+2) which shows it is not prime. On the other hand, the polynomial x^2+3x+5 is prime over the integers because we cannot factor it like the previous example. The only factors here are 1 and itself.

Factoring is handy to help determine roots of a polynomial. However, other tricks could be used such as the quadratic formula or using numerical approximation techniques.

In the past year Felipe watched 16 movies that he thought were very good. He watched 40 movies over the whole year. Of the movies he watched, what percentage did he rate as very good

Answers

Answer:

40%

Mark as Brainliest!

HELPPPPPPPPPPPP MEHHHHHHHHHHHHHH

Answers

Answer:

Translation

Step-by-step explanation:

A translation moves a shape up, down or from side to side but it does not change its appearance in any other way.

Here also, triangle ABC and triangle A'B'C' are equal only position is changed

5/7-(-1/7) plz help I am confused

Answers

Answer:

[tex] \frac{6}{7} [/tex]

Step-by-step explanation:

[tex] \frac{5}{7} - ( - \frac{1}{7} )[/tex]

[tex] \frac{5}{7} + \frac{1}{7} [/tex]

[tex] = \frac{6}{7} [/tex]

A case of 8 items costs $14.88 at costco. What is the price of each item

Answers

Answer: $1.86 per item

Explanation:

14.88 divided by 8

14.88/8= $1.86 per item
14.88/8 = $1.86 price of each item


A sailboat took 1/3 of an hour to travel
10 miles. At the same pace, how many
hours would the sailboat take to travel
120 miles?

Answers

The answer is C, 4 hours.

Answer:

4 hours

Step-by-step explanation:

if 1/3 of an hour = 10 miles

You need to get it to 120 miles.

120/10 = 12

And 12 x 1/3 = 4

I have a question if anybody answers the reward is brainlist
Whats 0÷0 ​

Answers

Answer:

0

Step-by-step explanation:

You can't divide anything by zero

0÷0=

=0

Mark all of the statements that are true.
O A. The domain for this function is the set {-5).
I B. The domain for this function is all real numbers.
C. This graph is not a function because the value of y is the same for
every value of x.
D. The range for this function is the set (-5).
I E. All real numbers are in the range of this function.

Answers

The correct statements are B and D
Any real value can be entered into the function but the answer will always be -5

We have the graph of a function (say f(x)).

We have to determine which of the given statements are correct w.r.t this graph of f(x).

What do you mean by domain and range of a function?

The domain of a function f(x) is the set of all values for which the function is defined, and the range of the function is the set of all values that f takes.

According to the question, we have the equation of this line as -

y = -5.

The value of y is constant for every value of x. Therefore, the range of the function is Set → {- 5}.

The function has same value → y= -5 for all values of x, ranging from →

- ∞ to + ∞, Therefore, its domain is the set of all Real numbers.

A function can have same value of y for different values of x.

Hence, Option B and D are correct.

To solve more questions on domain and range, visit the link below-

https://brainly.com/question/27242650

#SPJ2

Find the Sum
734,839 + 788,958

Answers

Answer:

1523797

Step-by-step explanation:

Have a good day, hope I helped

Answer:

1,523,797

Step-by-step explanation:

734,839 + 788,958

= 1,523,797

What number should go in the empty boxes to complete the calculation for finding the product of 0.53 × 0.67?

(Hint: it is the same number. Input only one number in the blank.)

will give (brainlist)

Answers

Answer:

1st box: 1

2nd box: 1

Step-by-step explanation:

1: 3 * 7 = 21, so only put the 1 and carry the 2.

2: 5 * 6 = 30, but we have a 1 from the previous answer given so 31, but only put the 1.

I hope this helps :)

Can you guys please help with these questions I’m extremely stuck. It’s algebra 2 inverse functions.

Answers

9514 1404 393

Answer:

The graph of a function and its inverse are reflections of each other in the line y=xFor each ordered pair (x, y) in a function, the corresponding ordered pair in the inverse function is (y, x)see the attachment for the tables of values

Step-by-step explanation:

Consider the function and its inverse shown in the second attachment. The function is ...

  f = {(a, 3), (b, 1), (c, 2)}

The inverse function is ...

  f^-1 = {(3, a), (1, b), (2, c)}

You will notice that the "x" and "y" of each pair are swapped between the function and its inverse. That is the point of an inverse function. It gives you the input that caused the function to create the output. For example, the pair (3, a) in the above inverse function tells you that the output 3 was the result of an input of 'a' to the original function.

__

On a graph, the swapping of input and output is essentially equivalent to relabeling the axes. If we keep the same x- and y-axis labels, it is equivalent to reflecting the function graph across the line y=x. That reflection is represented by the transformation (x, y) ⇒ (y, x).

__

The point of the exercise here is for you to identify some ordered pairs of one of the functions and realize that the inverse function reverses their order. You are to see that the graph of the function and its inverse are reflections of each other across the diagonal line y=x.

help on bothering please !!!!!reaangie nas perimeter
en
is in bana is with 25.
the value of m
En 182
Hn 224
Gn - 12.2
) -
The area of a rectangle is 134 + 2 square
units. The width is 9. The length is 6.
What is the length?
A 6.5
C 15.5
B 13
D 19

Answers

Answer:

6) F

7) B

Step-by-step explanation:

6)

[tex]\boxed{perimeter \: of \: rectangle = 2(length + breadth)}[/tex]

60= 2[½(n -6) +2n +5]

Expand:

60= 2(½)(n -6) +2(2n) +2(5)

60= n -6 +4n +10

Simplify:

60= 5n +4

Minus 4 on both sides:

5n= 60 -4

5n= 56

Divide both sides by 5:

n= 56 ÷5

n= 11.2

Thus, option F is correct.

7)

[tex]\boxed{area \: of \: rectangle = length \: \times breadth}[/tex]

13q +2= 9(q +6)

Expand:

13q +2= 9q +54

Bring q terms to one side, constant to the other:

13q -9q= 54 -2

Simplify:

4q= 52

Divide both sides by 4:

q= 52 ÷4

q= 13

Thus, option B is correct.

Help please will mark brainliest

Answers

Answer:

c no is correct pls mark me branilest

Answer:

(x, y) = (y, x). Exercise 1.3.4. Let X,Y,Z and W be nonempty sets. Then, prove the following statements

I need help with this Math Problem- (I got 7/8 inches more.) Planning to attach a plywood panel with nails 1 3/4 long.The panel is 3/8 inch thick.And beneath the panel is sheetrock which is 1/2 inch thick. How many inches of the nail should go into the wood frame?
Thks again

Answers

9514 1404 393

Answer:

  7/8 in  (you are correct)

Step-by-step explanation:

The amount of nail that goes into the framing member is ...

  1 3/4 -3/8 -1/2

  = 1 6/8 -3/8 -4/8

  = 1 -1/8 = 7/8 . . . . inches

Walter car gets 41 miles per gallon. Walter is planning a vacation which is 372 miles away. How many gallons of gas will Walter use for this round trip?

Answers

372 x 2 = 744
744/41 = 18.146 or 18.2 gallons of gas rounded
If they want a whole number 18.2 rounds down to 18.

!!!!!!!!!!!!!!!!!!!!!!!!!!!!!!!!!!helpppppppp

Answers

Answer:

2W + 56 = 78 , W = 10

Step-by-step explanation:

perimeter of rectangle = 2*the Length + 2*the Width

Is (1,3) a solution of y = 3x2?

Answers

No this is the awnser-3x+y=2

a) (-63) ÷ (-9)
b) (+72) ÷ (-8)

Answers

Answer:

A. +7

B. - 9

Hope this help you

How to numerically write five and eight then multiply by 4

Answers

Answer:

5 + 8 × 4

Assuming "five and eight" means 5 + 8

Get it right I promise I will give brainlist answer ASAP

Answers

I hope it helps. I hope it helps

Answer:

12 unit

I hope it helps

Find n so that mean of the data set is 140. (120, 145, 130, 80, 95, 100, 340, n)

Answers

Answer:

Answer: n is 110

Step-by-step explanation:

Formular:

[tex]mean = \frac{ \sum(n_{1}, \: n _{2}, \: ...n_{n} ) }{n} \\ [/tex]

therefore:

[tex]140 = \frac{(120 + 145 + 130 + 80 + 95 + 100 + 340 + n)}{8} \\ \\ (8 \times 140) = 1010 + n \\ 1010 + n = 1120 \\ n = 1120 - 1010 \\ n = 110[/tex]

Anyone know how to do this

Answers

Answer:

A B C D E MIGHT BE UR ANSWER

What is the slope of a line that passes through points (-1, 4) and (4, 9)?
-1
1
3/5
5/3

Answers

The right answer is 1

Find two positive numbers whose difference is 30 and whose product is 2584.

Answers

Answer:

[tex]38[/tex] and [tex]68[/tex].

Step-by-step explanation:

Let [tex]x[/tex] be the smaller one of the two number.

[tex]x[/tex] must be a positive integer. The other number would be [tex](x + 30)[/tex].

The question states that the product of the two numbers is [tex]2584[/tex]. In other words:

[tex]x\, (x + 30) = 2584[/tex].

Rearrange this equation and solve for [tex]x[/tex]:

[tex]x^{2} + 30\, x - 2584 = 0[/tex].

The first root of this quadratic equation would be:

[tex]\begin{aligned}x_{1} &= \frac{(-30) + \sqrt{30^{2} - 4 \times (-2584)}}{2} \\ &= \frac{(-30) + \sqrt{900 + 10336}}{2} \\ &= \frac{(-30) + \sqrt{11236}}{2} \\ &= \frac{(-30)}{2} + \sqrt{\frac{11236}{2^{2}}} \\ &= (-15) + \sqrt{2809} \\ &= (-15) + 53 \\ &= 38 \end{aligned}[/tex].

Similarly, the second root of this quadratic equation would be:

[tex]\begin{aligned}x_{1} &= \frac{(-30) - \sqrt{30^{2} - 4 \times (-2584)}}{2} \\ &= (-15) - 53 \\ &= -68\end{aligned}[/tex].

Since the question requires that both numbers should be positive, [tex]x > 0[/tex]. Therefore, only [tex]x = 38[/tex] is valid.

Hence, the two numbers would be [tex]38[/tex] and [tex](38 + 30)[/tex], which is [tex]68[/tex].

9514 1404 393

Answer:

  38 and 68

Step-by-step explanation:

Let x represent the average of the two numbers. Then their product is ...

  (x -15)(x +15) = 2584

  x^2 = 2584 +225 = 2809 . . . . . . use relation (x -a)(x +a) = x^2 -a^2; add a^2

  x = √2809 = 53

Then the two numbers are ...

  53 +15 = 68

and

  68 -30 = 38

The two numbers are 38 and 68.

_____

Additional comment

Solving the problem in this way is equivalent to writing the quadratic equation for the smallest (or largest) number and solving that by completing the square. The "solution" x = -53 is not relevant in this problem.

In 5×7 = 35 the numbers five and seven are called ? in the number 35 is called the ?

Answers

Answer:

5 and 7 are factors

35 is the product

Step-by-step explanation:

5 and 7 are the factors

you multiply factors to get a product

35 is a product of the factors, 5 and 7

If my answer is incorrect, pls correct me!

If you like my answer and explanation, mark me as brainliest!

-Chetan K

COME ON HELPPPPPPPPPPPPPPPPPPPPPPPPPPPPPPPPPPPPPPPP LOSING POINTSS HERE!!!!!

Answers

Answer:

The 1st one for the first pic and I'm not sure but c for the 2nd one

Answer:

a and c

Step-by-step explanation:

Other Questions
Which of the following is equal to the rational expression when x5 GIVING BRAINLY Explain why students excel according to Nature v Nurture debate/question?(must explain both sides) plzz help Add 4.2+(-0.4)plot the first addend and then the sum on the number line Which two types of energy does a book have as it falls to the floor?A. Kinetic energyB. Nuclear energyC. Light energyD. Magnetic energyE. Potential energy Need this worked out please so I can understand ita) Find the slope of the curve y= x^3 + 3 at the point P(-2, -5)b) Find an equation of the tangent line to the curve at P(-2, -5) Describe how scientific investigations lead to new scientific questions. 2. Why did the terrorists target the Twin Towers as part of the 9/11 attack? *O C. They had just been built and had been in the news a lot.O A. They represented America's economic power.O B. They contained offices for many government officials.O D. They were close to where the terrorists attended flight school. simplify -5^2-2^4 and it just says to make it 20 words long HELP ME!!! Steve goes on a road trip. He travels a total of 835 miles, and the trip takes 10 hours from start to finish.What is his average rate of change of distance? Please include units in your answer. The production of offsrpings is a charatericities of life that enables the continuation of a species. a.How was the life of our early ancestors? Which is an example of an Enlightenment idea? aFifth Amendment incrimination protection bFirst Amendment freedom of religion guarantees cFourth Amendment search and seizure protection dFirst Amendment freedom of speech guarantees One of the goals of mindfulness is to learn to respond wisely to things that happen to you True False Solve h + 6 / 5 = 2 Solve for h explain why the person, place, and time of the recording can affect value of a source to a historian 46xX =degrees. What are the possible downsides to wearable technology? Are there any ways to design around these issues? PLS HELP I'M STUCKKKK! :) WILL MARK BRAINLIEST! :) The Caravel ship, sextant, andare inventions related to seanavigationA. printing pressB. plowC. wagonD. compass Someone plz help me :(